6-10 divide, another onee thank you!!​

6-10 Divide, Another Onee Thank You!!

Answers

Answer 1

Answers:

10.) [tex]\displaystyle \pm{5}[/tex]

9.) [tex]\displaystyle 1\frac{1}{2}[/tex]

8.) [tex]\displaystyle \pm{1\frac{1}{2}}[/tex]

7.) [tex]\displaystyle \pm{1\frac{1}{2}}[/tex]

6.) [tex]\displaystyle \pm{\frac{1}{2}}[/tex]

Step-by-step explanations:

10.) [tex]\displaystyle \frac{\sqrt{200}}{\sqrt{8}} \hookrightarrow \sqrt{25} \hookrightarrow \frac{\pm{10\sqrt{2}}}{\pm{2\sqrt{2}}} \\ \\ \boxed{\pm{5}}[/tex]

9.) [tex]\displaystyle \frac{\sqrt[3]{135}}{\sqrt[3]{40}} \hookrightarrow \sqrt[3]{3\frac{3}{8}} \hookrightarrow \frac{3\sqrt[3]{5}}{2\sqrt[3]{5}} \\ \\ \boxed{1\frac{1}{2}}[/tex]

8.) [tex]\displaystyle \frac{\sqrt[4]{162}}{\sqrt[4]{32}} \hookrightarrow \sqrt[4]{5\frac{1}{16}} \hookrightarrow \frac{\pm{3\sqrt[4]{2}}}{\pm{2\sqrt[4]{2}}} \\ \\ \boxed{\pm{1\frac{1}{2}}}[/tex]

7.) [tex]\displaystyle \frac{\sqrt{63}}{\sqrt{28}} \hookrightarrow \sqrt{2\frac{1}{4}} \hookrightarrow \frac{\pm{3\sqrt{7}}}{\pm{2\sqrt{7}}} \\ \\ \boxed{\pm{1\frac{1}{2}}}[/tex]

6.) [tex]\displaystyle \frac{\sqrt{12}}{\sqrt{48}} \hookrightarrow \sqrt{\frac{1}{4}} \hookrightarrow \frac{\pm{2\sqrt{3}}}{\pm{4\sqrt{3}}} \\ \\ \boxed{\pm{\frac{1}{2}}}[/tex]

I am joyous to assist you at any time.


Related Questions

please help, what would the answer be?

Answers

27/3 = 9ft


3 x 2 = 6

Scale is 1:2 ratio

9 x 2 = 18ft

18 x 6 = 108ft

The answer is probably 108ft, there are no given scale ¯\_(ツ)_/¯

Please help me my teacher hasn't been helping me I really need your help

Answers

Answer:

550.5323531336038

Step-by-step explanation:

I have a hint: if you ever need help with triangle trigonometry questions go to carbside depot trigonometry calculator it is a literal life savor

When Derek planted a tomato plant, he expected to be picking his first ripe tomato in 45 days. His estimate was 25% less than the actual time. How long did it take before Derek picked his first ripe tomato?

Answers

Answer:

60 days.

Step-by-step explanation:

Let the actual time be x, then:

x - 0.25x = 45

0.75x = 45

x = 45/0.75

= 60 days.

HELP ME PLEASE
PRETTY PLEASEEEEEEEEEEE

Answers

Answer:

93

Step-by-step explanation:

19x-2 =  18x+3

x=5

15(5) -2=93

18(5)+3=93

If 3x - 5y = 11 and 2x + 3y = 5, then what is the ratio of x to y?

Answers

Answer:

-58/7

Step-by-step explanation:

Alright so this is a system of equations. First we'll solve the system, and then find the ratio afterwards.

[tex]3x-5y = 11\\2x+3y =5\\[/tex]

Isolate for y on both.

[tex]2x + 3y = 5\\3y = 5-2x\\y = \frac{5-2x}{3}[/tex]

and

[tex]3x - 5y = 11\\-5y = 11-3x\\y = \frac{11-3x}{-5}[/tex]

Set both equations equal to each other:

[tex]\frac{11-3x}{-5} = \frac{5-2x}{3}\\ \\\frac{3(11-3x)}{-5} = 5-2x\\ \\3(11-3x) = -5(5-2x)\\\\33 - 9x = -25 + 10x\\33 = -25 + 19x\\58 = 19x\\\frac{58}{19} = x[/tex]

We've got x, now let's solve for y:

[tex]y = \frac{11-3x}{-5} = \frac{11-3(\frac{58}{19}) }{-5}[/tex]

Now we got both x and y, and what they equal.

[tex]y = \frac{-7}{19}\\[/tex]

[tex]x = \frac{58}{19}[/tex]

The ratio of x to y, is essentially [tex]\frac{x}{y}[/tex]. So we will calculate that.

[tex]\frac{\frac{58}{19} }{\frac{-7}{19} } = \frac{58}{19} * \frac{19}{-7} = \frac{-58}{7}[/tex]

PLSS help important
(will gibe brainliest)!
I dont understand this

Answers

To form the vector notation for the translation:

   --> must find how much the new graph moved horizontally from the old

        graph

            --> the graph moved 8 units horizontally

   --> must find how much the new graph moved vertically from the old

        graph

           --> the graph moved 4 units vertically

In vector notation, that would be (8,4)

Hope that helps!

Is (-7, -3) a solution to this system of equations?

x = -7
y = -x - 10

Yes or no

Answers

Answer:   Yes

===========================================================

Explanation:

The point (-7,-3) means x = -7 and x = -3

Right off the bat, the first equation x = -7 is proven true based on the first coordinate.

Let's now plug the coordinates into the second equation.

y = -x-10

-3 = -(-7)-10

-3 = 7-10

-3 = -3

Which is a true statement.

Both equations are true when (x,y) = (-7,-3)

This is why it is a solution to the system. It turns out it's the only solution to this system. This system is consistent and independent.

You can use a graphing tool like Desmos to plot the two equations, and you should see them crossing at the point (-7,-3)

Answer:

The answer is yes

*View the attached graph to check your answer graphically.*

Step-by-step explanation:

x = -7

y = -x - 10

For this problem, I will be using substitution, since the second equation is already in the slope-intercept form.

First, I will substitute the first equation, for x, into the first equation:

x = -7

y = -x - 10

y = -(-7) - 10 <== multiplying two negatives, makes a positive

y = 7 - 10

y = - 3 <== the value of y

Now, we find the value of x by substituting - 3 for y:

y = -x - 10

- 3 = -x - 10

+10     +10

7 = -x <== you can't have a negative variable

/-1    /-1

-7 = x <== the value of x

(x, y) ==> (-7, -3)

Therefore, yes (-7,-3) is a solution to this system of equations.

*View the attached graph to check your answer graphically.*

Hope this helps!

how does the number if possible outcomes of a single event help you determine the total number of possible outcomes of a compound event?

Answers

The possible outcomes of an event is the sample space

The product of the sample spaces of the single events determines the total number of possible outcomes of a compound event

How to determine the possible outcomes?

Assume that the number of outcomes of n single events are:

[tex]n_1[/tex], [tex]n_2[/tex], [tex]n_3[/tex],........ [tex]n_n[/tex]

When these single events are combined, they form a compound event.

The number of outcomes of the compound event is the product of the number of outcomes of the single events.

i.e.

[tex]Outcomes = n_1 * n_2 * ......... n_n[/tex]

Hence, the product of the sample spaces of the single events determines the total number of possible outcomes of a compound event

Read more about outcomes at:

https://brainly.com/question/24756209

An investment account plays 3.7% annual interest compound weekly. If $370 is invested in this account, what will be the balance after 12 years?

Answers

Answer: 358

Explanation:  370 - 12 = 358

Sammy’s dad drove their car 150 miles in three hours. At this rate, how far would he drive in nine hours?

Answers

Answer:

distance in 3 hours = 150 miles

distance in 1 hour = 150/3 = 50 miles

distance in 9 hours = 50 × 9 = 450 miles

Answer:

Sammy's dad traveled 450 miles in 9 hours.

Step-by-step explanation:

mph = [tex]\frac{Miles}{Hours}[/tex]

Miles = 150

Hours = 3

Plug into the formula of distance/time

[tex]\frac{150}{3} = 50mph[/tex]

Sammy's dad is driving at 50mph

In three hours you can use this formula:  [tex]50mph=\frac{miles}{9}[/tex]

Multiply both sides by 9:  [tex]50mph * 9 =\frac{miles}{9} *9[/tex]

Solve:

[tex]50mph * 9 =miles[/tex]

[tex]450 =miles[/tex]

Sammy's dad traveled 450 miles in 9 hours.

Hope this helped! :)

Hazel and 4 of her friends bought tickets for a baseball game. They received a $30 group discount. If the total cost was under $125, how much could each ticket have been?

Answers

Each ticket could have cost up to $31.

What is the discount?

A discount is a reduction in the price of an item or service. It is often offered as an incentive to encourage customers to purchase or use a particular product or service. The discount amount may be a percentage of the total price, a fixed dollar amount, or a combination of both.

Let the cost of each ticket be $x. Then the total cost for 5 tickets would be 5x. After applying the $30 group discount, the total cost would be 5x - $30.

We know that the total cost was under $125, so we can set up the inequality:

5x - $30 < $125

Simplifying this inequality, we get:

5x < $155

Dividing both sides by 5, we get:

x < $31

Hence, each ticket could have cost up to $31.

To learn more about the discount, visit:

https://brainly.com/question/1548141

#SPJ1

PRESLEY
You are curious about the amount of coffee that the giant fountain cup could actually hold.
The dimensions of the fountain cup are as follows:
• Height is 8 feet.
• Diameter of the top is 6 feet.
• Diameter of the base is 4 feet.
Use this formula with height, h, radius of the base, roase, and radius of the top, rtop, to determine the volume
of the cup:
(πh).
V= -((rbase)+ (rbase) (stop) + (top)).
3
There are 7.5 gallons of liquid per cubic foot.
Enter the volume, in gallons, of the fountain cup.

Answers

The volume of the fountain cup in gallons is 4775.2 gallons

Volume of a frustum

Since the fountain cup is in the shape of a frustum, its volume is given by

V = πh/3(r² + rr' + r'²) where

h = height of cup = 8 feet, r = radius of base of cup = 4 feet and r' = radius of top of cup = 6 feet.

So, substituting the values of the variables into th equation, we have

V = πh/3(r² + rr' + r'²)

V = π × 8 ft/3[(4 ft)² + 4 ft × 6 ft + (6 ft)²]

V = π × 8 ft/3[16 ft² + 24 ft² + 36 ft²]

V = π × 8 ft/3 × (76 ft²)

V = 608π ft³/3

V = 1910.088 ft³/3

V = 636.69 ft³

V ≅ 636.7 ft³

Volume of the fountain cup in gallons

Since there are 7.5 gallons per cubic foot,

The volume of the fountain cup in gallons is V = 636.7 ft³ × 7.5 gallons/ft³ = 4775.2 gallons

So, the volume of the fountain cup in gallons is 4775.2 gallons

Learn more about volume of a frustum here:

https://brainly.com/question/14268491

-2x^2+bx -5 Determine the b-value that would ensure the function has two real root.

Answers

Answer:

No solution

Step-by-step explanation:

Given is the quadratic function

y = -2x² + bx - 5

In order to have two real roots the discriminant should be posivive

D = - b² - 4acD = - b² - 4(-2)(-5) = - b² - 40

We need D > 0

-b² - 40 > 0b² + 40 < 0b² < - 40

There is no solution as b² is never negative

Which graph matches? I'll give brainliest. and 100pts

Answers

Function:

[tex]\sf 2\left(3\right)^{x}+2[/tex]

Find y-intercept:

[tex]y=2\left(3\right)^{0}+2[/tex]

[tex]y=\sf 4[/tex]

option B is correct as it cuts y-intercept at 4

Answer:

This is a positive exponential graph with a y-intercept of 4 (when x is 0, y is 4)

Hope this helps!

6,402.66 divided by 459438.26

Answers

Answer: 0.0139358  

Step-by-step explanation: 999,999,999,999,999,999,999,999,999,999,999% sure!

On Saturday, the temperature was
75.5°F. The temperature rose by
6°F on Sunday, then dropped by
3.5°F on Monday. Write an expression
to represent how the temperature
changed. What was the temperature
on Monday?

Answers

Answer:

78°F

Step-by-step explanation:

75.5°F on Saturday...

75.5 + 6 = 81.5 on Sunday

81.5 - 3.5 = 78°F on Monday

The expression that represents the temperature change is

N = L + 6°F + 3.5°F

The temperature on Monday is 85°F.

What is an expression?

An expression is a way of writing a statement with more than two variables or numbers with operations such as addition, subtraction, multiplication, and division.

Example: 2 + 3x + 4y = 7 is an expression.

We have,

Temperature:

Saturday = L= 75.5°F ______(1)

Sunday = M = Rose by 6°F

M = L + 6 _____(2)

Monday = N = Dropped by 3.5°F

N = L + 6 - 3.5 _____(3)

The expression that represents the temperature change:

From (1), (2), and (3) we get

N = L + 6°F + 3.5°F

The temperature on Monday:

N = 75.5 + 6 + 3.5

N = 85°F

Thus,

The expression that represents the temperature change is

N = L + 6°F + 3.5°F

The temperature on Monday is 85°F.

Learn more about expressions here:

https://brainly.com/question/3118662

#SPJ5

A plant starts with 1 branch. Every year,
each branch becomes 3 branches. A
sketch of the plant for the first 3 years is
shown. How many branches will the plant
have in year 10?
TYP
I V
Year 1
Year 2
Year 3
How many branches would the plant
have in year 10 if the plant ad
5 branches the first year? (Each branch
still becomes 3 branches every year.

Answers

Answer:

120

Step-by-step explanation:

we go add 3+1=4 and then 4×3=12 and then 12×10=120

Find the distance between the two points

☟ ︎Photo down below ☟

Answers

[tex]\qquad\qquad\huge\underline{\boxed{\sf Answer☂}}[/tex]

Let's use distance formula ~

[tex]\qquad \sf  \dashrightarrow \: d = \sqrt{(x2 - x1) {}^{2} + (y2 - y1) {}^{2} } [/tex]

[tex]\qquad \sf  \dashrightarrow \: d = \sqrt{(2 - ( - 4)) {}^{2} + (0 - 1) {}^{2} } [/tex]

[tex]\qquad \sf  \dashrightarrow \: d = \sqrt{(2 + 4) {}^{2} + ( - 1) {}^{2} } [/tex]

[tex]\qquad \sf  \dashrightarrow \: d = \sqrt{ {}^{} 36+ 1{}^{} } [/tex]

[tex]\qquad \sf  \dashrightarrow \: d = \sqrt{ {}^{} 37{}^{} } [/tex]

Therefore, the required distance is [tex]\sf \sqrt{37}[/tex] units

Given the equation 47=38+n, tell which value is a solution from the solution set n={7, 9, 11, 13}

Answers

hello!

First, let's solve this equation by subtract 38 from both sides:-

[tex]\bigstar[/tex] 47-38=n

hence, the answer is:-

9=n

or

[tex]\bigstar{\underline{\boxed{\pmb{n=9}}}[/tex]

note:-

Hope everything is clear; if you need any clarification/explanation, kindly let me know, and I'll comment and/or edit my answer :)

How do you multiplying or dividing a fractions to obtain equivalent

Answers

To obtain an equivalent fraction by multiplying, multiply the whole fraction by any single number. Example: 3/4 multiplied by 3 is 9/12. Those fractions are equivalent.

To obtain an equivalent fraction by dividing, look at the top number and the bottom number. If they both can be divided by a number without it being a decimal, it’s an equivalent fraction. Example: 8/10, both of those numbers can be divided by 2 that isn’t a decimal, 4/5. 4/5 and 8/10 are equivalent

Solve for x: √(32^0 + 2/3)=(0.6)^2-3x​

Answers

Step-by-step explanation:

Given: √{32⁰ + (2/3)} = (0.6)²⁻³ˣ

Asked: Find the value of x = ?

Solution: Given that √{32⁰ + (2/3)} = (0.6)²⁻³ˣ

⇛√{1 + (2/3)} = (0.6)²⁻³ˣ

⇛√{(1/1) + (2/3) = (0.6)²⁻³ˣ

⇛√{(1*3 + 2*1)/3} = (0.6)²⁻³ˣ

⇛√{(3 + 2)/3} = (0.6)²⁻³ˣ

⇛√(5/3) = (0.6)²⁻³ˣ

Squaring on both sides then

⇛{√(5/3)}² = {(0.6)²⁻³ˣ}²

⇛√(5²/3²) = {0.6}(²⁻³ˣ)²

⇛√{(5*5)/(3*3)} = {0.6}(²⁻³ˣ)²

⇛5/3 = {0.6}(²⁻³ˣ)²

[[tex]\mathsf{\because}[/tex] (aᵐ)ⁿ = aᵐⁿ]

⇛5/3 = (0.6)²*²⁻³ˣ*²

⇛5/3 = (0.6)⁴⁻⁶ˣ

⇛5/3 = (6/10)⁴⁻⁶ˣ

⇛5/3 = {(6÷2)/(10÷2)})⁴⁻⁶ˣ

⇛5/3 = (3/5)⁴⁻⁶ˣ

⇛(3/5))⁻¹ = (3/5)⁴⁻⁶ˣ

[[tex]\mathsf{\because}[/tex] a⁻ⁿ = 1/aⁿ]

Base are the same, so the exponents must be equal.

[tex]\mathsf{\therefore}[/tex] -1 = 4 - 6x

Shift the number 4 from RHS to LHS, changing it's sign.

⇛-1 - 4 = -6x

⇛-5 = -6x

⇛x = {(-5)/(-6)}

[tex]\mathsf{\therefore}[/tex] x = 5/6

Answer: Hence, the value of x for the given problem is 5/6.

also read similar questions: (1/3)^-2 + (1/4)^-2 = (125)^x Find the value of x? https://brainly.com/question/25746379?referrer

(3^-2 × 4^-2)^-3 = 12^x In the given expression find the value of x. https://brainly.com/question/25746273?referrer

What is the number with the mark

Answers

[tex]\huge\mathsf\blue{♧ANSWER♧}[/tex]

[tex]\huge \mathfrak \blue{\frac{ - 7}{8} }[/tex]

In ΔWXY, y = 8.6 cm, x = 8.5 cm and ∠X=100°. Find all possible values of ∠Y, to the nearest 10th of a degree.

Answers

The possible value of ∠Y to the nearest tenth of a degrees in triangle WXY is 85.1 degrees.

How to find angle of a triangle using sine rule

Let's find angle Y using sine rule,

Hence,

x / sin X = y / sin Y

where

x = 8.5 cmy = 8.6 cm∠X = 100°

8.5 / sin 100° = 8.6 / sin Y

cross multiply

8.5 sin Y = 8.6 sin 100°

sin Y = 8.6 sin 100° / 8.5

sin Y = 8.4693466759 / 8.5

Y = sin ⁻¹ 0.99639372657

Y = 85.1325941735

∠Y = 85.1°

learn more on triangle here: https://brainly.com/question/23998296

Put the equation below into vertex form.

y=(x+9)(x+25)

Answers

Answer: (-17,-24)
Explanation: y= x^2+34x+225
a=1, b=34
X= - 34/2+1
x = -17
Y= (x+9)(x+25), x=-17
Y=-64
So vertex is (-17,-64)
If you put the equation into standard form and use point of symmetry formula to find the vertex you can answer the problem.

standard form:
y = ax^2 + bx + c
y = x^2 + 34x + 225
a=1 b=34 c=225

point of symmetry formula:
x = -b/2a
x = -34/2
x = -14

plug in x to find the vertex

y = (-14+9)(-14+25)
y = (-5)(11)
y = -55

vertex = (-14,-55)

vertex form
y = a(x-h)^2 + k [where (h,k) is the vertex]

so your answer is:

y = 1(x-(-14))^2 + (-55)

or

y = (x+14)^2 - 55


I hope this helps and is the BRAINLIEST!!

Good luck with your studies!

Lita must find the area of the sector enclosed by central angle QCR in circle C.

Points Q and R lie on circle C. The measure of angle Q C R is 74 degrees and the radius of circle C is 1 foot.


What steps should Lita take to correctly solve this problem?

A: Lita knows that m∠QCR=74∘ and that the whole circle has a measure of 360∘, so the portion of the area she wants to find is 74∘360∘. Therefore, she must multiply 74∘360∘ times the area, which is πr2.Lita knows that m∠QCR=74∘ and that the whole circle has a measure of 360 degrees textsf comma so the portion of the area she wants to find is 74∘360∘. Therefore, she must multiply 74∘360∘ times the area, which is πr2.

B: Lita knows that m∠QCR=74∘ and that the whole circle has a measure of 360∘, so the portion of the area she wants to find is 74∘360∘, so she must multiply 74∘360∘ times the area, which is 2πr.Lita knows that m∠QCR=74∘ and that the whole circle has a measure of 360 degrees textsf comma so the portion of the area she wants to find is 74∘360∘, so she must multiply 74∘360∘ times the area, which is 2πr.

C: Lita knows that m∠QCR=74∘ and that the whole circle has a measure of 360∘, so she should subtract 74∘ from 360∘. The portion of the area she wants to find is 286∘360∘, so she must multiply 286∘360∘ times the area, which is 2πr.Lita knows that m∠QCR=74∘ and that the whole circle has a measure of 360 degrees textsf comma so she should subtract 74 degrees from The portion of the area she wants to find is 286∘360∘, so she must multiply 286∘360∘ times the area, which is 2πr.

D: Lita knows that m∠QCR=74∘ and that the whole circle has a measure of 360∘, so she should subtract 74∘ from 360∘. The portion of the area she wants to find is 286∘360∘, so she must multiply 286∘360∘ times the area, which is πr2.

Answers

To find the area of the sector, Lita must multiply 74°/360° times the area of the circle, which is πr² (Option A).

What is the Area of the Sector of a Circle?

Area of sector = ∅/360 × πr², where ∅ = central angle, and r = radius of the circle.

Given the following:

∅ = m∠QCR = 74°r = 1 ft.

The area of the circle she wants to find is: 74°/360°.

Since the area of the whole circle is, πr², therefore, to find the area of the sector, Lita must multiply 74°/360° times the area of the circle, which is πr² (Option A).

Learn more about area of sector on:

https://brainly.com/question/22972014

round to the nearest hundredth
(please help picture inculded) ​

Answers

Answer:

AB ≈ 6.53

Step-by-step explanation:

using the cosine ratio in the right triangle

cos40° = [tex]\frac{adjacent}{hypotenuse}[/tex] = [tex]\frac{AC}{AB}[/tex] = [tex]\frac{5}{AB}[/tex] ( multiply both sides by AB )

AB × cos40° = 5 ( divide both sides by cos40° )

AB = [tex]\frac{5}{cos40}[/tex] ≈ 6.53 ( to the nearest hundredth )


I need the help with geometry problem

Answers

Answer: sqrt(182)

Step-by-step explanation:

please help me

Select the reason that best
supports Statement 4 in the
given proof.
A. Transitive Property
B. Multiplication Property of Equality
C. Substitution
D. Given

Answers

Answer: C

Step-by-step explanation:

They are substituting PQ=25 from steps 1 and 3.

Write the following series in sigma notation.
7 + 16 + 25 +34 +43 +52 + 61

Answers

The series 7 + 16 + 25 +34 +43 +52 + 61 is an illusration of arithmetic series

The sigma notation of the series is: [tex]\sum\limits^7_{n=1} {9n - 2}[/tex]

How to write the series in sigma notation?

The series is given as:

7 + 16 + 25 +34 +43 +52 + 61

The above series is an arithmetic series, with the following parameters

First term, a = 7Common difference, d = 9Number of terms, n = 7

Start by calculating the nth term using:

a(n) = a + (n - 1) * d

This gives

a(n) = 7 + (n - 1) * 9

Evaluate the product

a(n) = 7  - 9 + 9n

Evaluate the difference

a(n) = 9n - 2

So, the sigma notation is:

[tex]\sum\limits^7_{n=1} {9n - 2}[/tex]

Read more about arithmetic series at:

https://brainly.com/question/6561461

Which value is a solution to x <

Answers

Answer:

0

Step-by-step explanation:

it less than 4 so u can't pick 4,so it is any number less than 4

Answer:

B) 0

Step-by-step explanation:

The open circle on 4 and the arrow pointing to the left means x < 4

x < 4 means that x is smaller than 4

4 = 4   incorrect as this says 4 is equal to 4

0 < 4   CORRECT as zero is smaller than 4

5 > 4   incorrect as 5 is bigger than 4

4.5 > 4   incorrect as 4.5 is bigger than 4

Other Questions
What is the best summary of the selection a legacy of stone 16 of the fruits at a warehouse were apples and the remaining fruits were oranges. There were 291 more red apples than green apples and there were 3455 oranges in the warehouse. How many red apples were there? I need information on Oceania to increase my grade. HELP! The area enclosed by the graphs of y = 1/x, y = 1, and x = 3 is rotated about the line y = -1. Find the volume and show steps. In an area of East Texas, the most intense competition would most likely occur between white-tailed deer andA. brown bearsB. squirrelsC. other white-tailed deerD. coyotes Shaun oversees the marketing and sales activities for a company that sells products through its website. his job title is best described as . nadine analyzes ways to improve the number of viewers and time they spend on a website by promoting the site and updating its text and graphics to improve its search rankings. her job title is best described as . What active reading stragery inviles the readers best guses about what will happen next in the text You need to buy insulation to cover the inside of a shipping container. The container is a rectangular prism that is 15 feet long, 8 feet wide, and 7 feet high. How much insulation do you need to buy if you want to insulate all the inside surfaces of the shipping container except the floor? A square rug has an area of 256 square feet. What is the perimeter of the rug? gi i need help with this question if you can help me asap that whould be great The ratio of the number of dolls Jacky had to the number of dolls Peter had was 5:2 but, after Jacky gave 15 dolls to Peter, they have an equal number of dolls. How many dolls did they have altogether? Simplify: 14-30/ 2 (- 4)Thanks! :) What is the temperature of the flame in degrees Celsius?You measure the flame of a candle and get a reading of 1,376.06 K. This value is equal to What is kings view of the disorder that is occurring as a result of civil rights demonstrations? it is blocking social progress. It is desirable, as african americans have been suffering injustice for generations it is necessary to the eventual defeat of injustice Which of the following was not an invention of the Han period?A. PorcelainB. WheelbarrowC. PaperD. Silk Charmaine got a prepaid debit card with $15 on it. for her first purchase with the card, she bought some bulk ribbon at a craft store. the price of the ribbon was 21 cents per yard. if after that purchase there was $12.27 left on the card, how many yards of ribbon did charmaine buy? What was the name of the first satellite launched into orbit by russia in 1957?. PLEASE HELP!! Need by 3/29/22!Find the area of these shapes Hello people ~Water cannot be used as a fire extinguisher to put out A. Burning charcoalB. Burning clothC. Burning oilD. Burning wood The belief of lack of control might lead to cardiomyopathy (heart weakness).True or False?